LSAT and Law School Admissions Forum

Get expert LSAT preparation and law school admissions advice from PowerScore Test Preparation.

 Administrator
PowerScore Staff
  • PowerScore Staff
  • Posts: 8917
  • Joined: Feb 02, 2011
|
#38402
Complete Question Explanation
(The complete setup for this game can be found here: lsat/viewtopic.php?t=15027)

The correct answer choice is (B)

The question stem establishes that Q is immediately before R, creating a new QR Block. The block’s placement is not particularly restricted: there are four possible ways to position that block in our setup. The same holds true for the VW Block, creating 8 possible configurations of these two blocks (not accounting for T and S). If something looks like too much work, it probably is!

Although the precise location of either block is unknown, notice that the position of P makes it necessary to place one of the blocks to the “left” of P, and the other—to the “right” of P:
PT76_O15 LG Explanations_Game #1_#4_diagram 1.png
The question is only concerned with the location of S—where can S not be? Given the location of our blocks, we can easily prephrase an answer to that question: S cannot be either 2 or 6. This agrees with answer choice (B), which is correct.

Always know what the question is asking you to do, and never do more than the minimum amount of work necessary to answer that question!
You do not have the required permissions to view the files attached to this post.

Get the most out of your LSAT Prep Plus subscription.

Analyze and track your performance with our Testing and Analytics Package.